4
$\begingroup$

Let $n$ be a positive integer, $p$ a (positive rational) prime, and $\mathbb K$ an algebraically closed field. If ${\rm char}(\mathbb K) = 0$ then ${\rm GL}_n(\mathbb K)$ is divisible (see here). But what about ${\rm SL}_n(\mathbb K)$? Again, the answer is negative if $\mathbb K$ has finite characteristic, and again we may assume that (i) our matrices are in Jordan normal form and (ii) $\mathbb K$ is the complex field, thanks to the Lefschetz principle and the fact that, by Laplace's formula, the determinant of an $n$-by-$n$ "formal matrix" can be expressed as a wff in the first-order language $\mathcal L = (+, \cdot, -, 0, 1)$ of (the theory of) rings. Thus, let us consider a Jordan matrix of size $n \times n$, say $J = {\rm diag}(J_1, \ldots, J_m)$, where $J_i$ is a Jordan block of size $k_i \times k_i$. If $\det(J) = 1$ and $p \nmid \gcd(k_1, \ldots, k_m)$, it is not difficult to prove that there exists $A \in {\rm SL}_n(\mathbb C)$ such that $A^p = J$.

Question 1. What about the other cases?

The problem should be well-known and I feel that the answer is in the negative, but so far I couldn't either get a reference or find a counterexample by myself. EDIT 2. The $2$-by-$2$ Jordan block with eigenvalue $-1$ is a counterexample, and in some sense it is the only one possible for $n = 2$ (see the comments below). Thus, it seems natural to ask the following:

Question 2. (i) Is there any "explicit characterization" of those matrices in ${\rm SL}_n(\mathbb C)$ which fail to have a $p$-th root for each prime $p$? (ii) In particular, is the set of these matrices the union of a finite number of conjugacy classes of ${\rm SL}_n(\mathbb C)$?

Let me rely on your common sense for the actual meaning to give to the expression "explicit characterization". Last but not least:

Question 3. Does anyone know where to find a reference for this kind of questions (concerned with the divisibility of specific subgroups of ${\rm GL}_n(\mathbb K)$ when $\mathbb K$ is an algebraically closed field, either in characteristic zero or not)?

Thanks in advance for any help.

$\endgroup$
2
  • 2
    $\begingroup$ Did you try the obvious place to start, namely a $2\times 2$ Jordan block with $-1$s on the diagonal? $\endgroup$
    – user30035
    Mar 21, 2013 at 11:17
  • $\begingroup$ Ehr... Yes, but somehow I missed this "limit case" while working to something slightly more general: If $J = \left[\begin{array}{cc} \lambda & \mu \\ 0 & \lambda^{−1}\end{array}\right]$ for some $\lambda \in \mathbb C^\times\setminus\{−1\}$ and $\mu \in \mathbb C$, then $\left[\begin{array}{cc} a & c \\ 0 & b\end{array}\right]^2=J$ for $a=|\lambda|^{1/2}e^{i\frac{\theta}{2}}$, $b=a^{−1}$ and $c=(a+b)^{−1} \mu$, where $\theta$ is (the principal value of) the complex argument of $\lambda$. I edited the OP, fixed my mistake, and updated Q2. Thanks! $\endgroup$ Mar 21, 2013 at 13:31

1 Answer 1

5
$\begingroup$

$SL_n(k)$ is not divisible over any algebraically closed field $k$ and $n>1$.

This would be indeed a nice exercise in a Linear Algebra course.

Proof: Let $\zeta\neq 1$. I'll show that there is no $X\in SL_n(k)$ such that $X^n=A$ for
$$A=\scriptstyle\begin{pmatrix} \zeta & 1 & & & \newline & \zeta & 1 & & \newline & & \ddots & \ddots & \newline & & & \zeta & 1 \newline & & & & \zeta \end{pmatrix}.$$

Let $X^n=A$ and let $\mu_1,...,\mu_n$ be the eigenvalues of $X$. Clearly $\mu_i^n=\zeta$. Each eigenvector of $X$ is also an eigenvector of $A$ and since $A$ has exactly one eigenvector (up to scalar multiples), we conclude $\mu_1=\ldots=\mu_n$. Hence $\det(X)=\mu_1^n=\zeta\neq 1$. qed.

$\endgroup$
2
  • $\begingroup$ Clear and very nice. I really wonder if one can cluster all the matrices in ${\rm SL}_n(\mathbb C)$ which don't have a $p$-th root for some prime $p \le n$ in a finite number of conjugacy classes: Your example is still very particular, which tempts me to think that there may be only "few" exceptions. $\endgroup$ Mar 21, 2013 at 21:26
  • 1
    $\begingroup$ Of course this example requires $\zeta^n = 1$ (or else $A \not\in \operatorname{SL}_n(k)$), and so I think doesn't work when $n$ is the characteristic of $k$. $\endgroup$
    – LSpice
    Oct 23, 2015 at 2:20

Your Answer

By clicking “Post Your Answer”, you agree to our terms of service and acknowledge you have read our privacy policy.

Not the answer you're looking for? Browse other questions tagged or ask your own question.